781 final to study

Lakukan tugas rumah & ujian kamu dengan baik sekarang menggunakan Quizwiz!

You are a nurse working in a community mental health clinic. You receive a call from a known patient being treated with Clozaril for schizophrenia. Which of the following statements would require an immediate response?

"I think I'm coming down with the flu." d/t low WBC count

A nurse is conducting a one-to-one session with a patient. The patient states, "Shhh...can't you hear him? It's the devil. He's talking to me." The nurse's best response is:

"That sounds very scary, but I don't hear it."

Prodromal Phase

*begins to show signs of significant deterioration in functioning* - can be brief but average length is 2-5 years - social withdrawal - cognitive impairment - some adolescents develop sudden onset of OCD behavior - 50% c/o depressive symptoms goal is to promote early recognition of behavior & early interventions to improve long term outcomes

premorbid phase

*signs that occur before clear evidence of illness* - Social maladjustment: Shy, quiet, passive, withdrawn, avoiding anything that brings attention to them - asocial, antisocial, avoidance of team sports & dating - Poor peer relationships - Doing poorly in school goal is to prevent transition of illness

Nihilistic delusion

+ symptom False idea that the self, a part of the self, others or the world is nonexistent or has been destroyed ex. believing life is meaningless or belief that they do not exist

Delusion of reference

+ symptom Fixed false beliefs in spite of evidence to the contrary

Delusion of control/influence

+ symptom Pt believes certain objects or persons have control over his/her behavior

Delusion of persecution

+ symptom belief that others intend harm

Somatic delusion

+ symptom false idea about functioning of body ex. 80 y/o believes she is pregnant

delusion of grandeur

+ symptom •An exaggerated feeling of importance, power, knowledge, or identity •"I'm Jesus." I'm the president"

What is a leading theory of how neuronal activity and neurotransmitters are implicated in addiction?

- Addictive substances usurp normal limbic reward pathways, producing greater firing than usual reward pathways, producing greater firing than usual rewards experienced in day-to-day survival relevant functioning

Applicability to Nursing: Psychodynamic Theories - Important concepts:

- Interpersonal relationships - Defense mechanisms - Transference - Countertransference - Internal objects A therapeutic interpersonal relationship: core of psychiatric-mental health nursing intervention

Factors associated w/more positive outcome

- later age of onset - female - high-level pre-morbid functioning - abrupt onset of symptoms w/obvious predisposing factor (as opposed to gradual) - associated mood disturbance - rapid resolution of active phase symptoms - minimal residual symptoms - no family history - absence of structural brain abnormalities or normal neurological functioning *complete return to premorbid functioning is highly unlikely*

medical conditions associated with catatonia

- metabolic disorders (hepatic encephalopathy, DKA, hypo/hyperactive thyroidism, hypo/hyperadrenalism, hypercalcemia, vitamin B12 deficiency) - neurological conditions (epilepsy, tumors, cerebrovascular disease, trauma, encephalitis)

DSM-V Criteria

- must rule out other PMH disorders - must have negative s/s - must have 2 of following: delusions, hallucinations, disorganized speech, catatonic behavior, or grossly disorganized - level of functioning impaired in 1 or more major areas in life (ex. social, occupational, etc.) - continuous s/s persist for at least *6 months*

Residual Phase

- phase 4 periods of remission and exacerbation following active phase - some may keep negative symptoms - residual impairment often increases w/additional episodes of active psychosis (kindling effect)

When can alcohol related seizures occur?

1-2 days after the last drink

A client is in the active phase of paranoid schizophrenia. Which nursing intervention would aid in facilitating other interventions? 1. Assign consistent staff members. 2. Convey acceptance of the delusional belief. 3. Help the client understand the connection between anxiety and hallucinations. 4. Encourage participation in group activities.

1. Individuals with paranoia have extreme suspiciousness of others and their actions. It is difficult to establish trust with clients experiencing paranoia. All interventions would be suspect. Only by assigning consistent staff members would there be hope to establish a trusting nurse-client relationship and increase the effectiveness of further nursing interventions.

A client diagnosed with schizophrenia is brought to the emergency department by a family member. The client is experiencing social withdrawal, flat affect, and impairment in role functioning. To distinguish whether this client is in the prodromal or residual phase of schizophrenia, what question would the nurse ask? 1. "Has this client recently experienced an exacerbation of the signs and symptoms of schizophrenia?" 2. "How long have these symptoms been occurring?" 3. "Has the client had a change in mood?" 4. "Has the client been diagnosed with any developmental disorders?"

1. It is important for the nurse to know if this client has recently experienced an active phase of schizophrenia to distinguish the symptoms presented as indications of the prodromal or residual phase of schizophrenia. Schizophrenia is characterized by periods of remission and exacerbation. A residual phase usually follows an active phase of the illness. Symptoms during the residual phase are similar to those of the prodromal phase, with flat affect and impairment in role function being prominent.

A client newly admitted to an in-patient psychiatric unit is scanning the environment continuously. Which nursing intervention is most appropriate to address this client's behavior? 1. Offer self to build a therapeutic relationship with the client. 2. Assist the client to formulate a plan of action for discharge. 3. Involve the family in discussions about dealing with the client's behaviors. 4. Reinforce the need for medication compliance on discharge

1. The client described in the question is exhibiting signs of paranoia. Clients with this symptom have trouble trusting others. The nurse should use the therapeutic technique of offering self to assist in building a trusting therapeutic relationship with this client.

A new graduate nurse is assessing a 20-year-old client in the emergency department. The client is seeing and hearing things that others do not see or hear. The nurse tells the supervisor, "I believe the client has schizophrenia." Which of the following supervisor responses is the most appropriate? Select all that apply. 1. "How long has the client experienced these symptoms?" 2. "Has the client taken any drug or medication that could cause these symptoms?" 3. "It is not within your scope of practice to assess for a medical diagnosis." 4. "Does this client have any mood problems?" 5. "What kind of relationships has this client established?"

1. The duration of symptoms is an important finding to assess to determine the diagnosis of schizophrenia. One of the DSM-IVTR criteria is that symptoms need to be present for a significant amount of time during a 1-month period and last for 6 months. 2. A substance or general medical condition exclusion is an important finding to assess to determine the diagnosis of schizophrenia. One of the DSM-IV-TR criteria is that the presenting symptoms are not due to the direct physiological effects of the use or abuse of a substance or medication. 4. The presence of mood disorders is an important finding to assess to determinethe diagnosis of schizophrenia. Schizoaffective disorder and mood disorder with psychotic features must be ruled out for the client to meet the criteria for this diagnosis. No major depressive, manic, or mixed episodes should have occurred concurrently with the active-phase symptoms. If mood episodes have occurred during the active-phase symptoms, their total duration should have been brief, relative to the duration of the active and residual periods. 5. The ability to form relationships is an important finding to assess to determine the diagnosis of schizophrenia. One of the DSM-IV-TR criteria for this diagnosis is a disturbance in one or more major areas of functioning, such as work, interpersonal relationships, or self-care. When the onset is in adolescence, there should be a failure to achieve expected levels of interpersonal or academic functioning.

The nurse is assessing a client diagnosed with disorganized schizophrenia. Which symptoms should the nurse expect the client to exhibit? 1. Markedly regressive, primitive behavior, and extremely poor contact with reality. Affect is flat or grossly inappropriate. Personal appearance is neglected, and social impairment is extreme. 2. Marked abnormalities in motor behavior manifested in extreme psychomotor retardation with pronounced decreases in spontaneous movements and activity. Waxy flexibility is exhibited. 3. The client is exhibiting delusions of persecution or grandeur. Auditory hallucinations related to a persecutory theme are present. The client is tense, suspicious, and guarded, and may be argumentative, hostile, and aggressive. 4. The client has a history of active psychotic symptoms, but prominent psychotic symptoms are currently not exhibited.

1. When a client exhibits markedly regressive and primitive behavior, and the client's contact with reality is extremely poor, he or she is most likely to be diagnosed with disorganized schizophrenia. In this subcategory, a client's affect is flat or grossly inappropriate. Personal appearance is neglected, and social impairment is extreme.

Which clients respond with greater efficacy to antipsychotic drugs? Select all that apply. 1. A client with apathy 2. A client with delusions 3. A client with a loss of drive 4. A client with hallucinations 5. A client with a poverty of ideas

2, 4 delusions & apathy

On an in-patient unit, the nurse is caring for a client who is assuming bizarre positions for long periods of time. To which diagnostic category of schizophrenia would this client most likely be assigned? 1. Disorganized schizophrenia. 2. Catatonic schizophrenia. 3. Paranoid schizophrenia. 4. Undifferentiated schizophrenia.

2. A client diagnosed with catatonic schizophrenia exhibits marked abnormalities in motor behavior manifested in extreme psychomotor retardation with pronounced decreases in spontaneous movements and activity. Waxy flexibility is exhibited. Waxy flexibility is a type of posturing or voluntary assumption of bizarre positions in which the individual may remain for long periods. Efforts to move the individual may be met with rigid bodily resistance. The client described in the question is exhibiting signs and symptoms of catatonic schizophrenia.

A nurse is working with a client diagnosed with schizoid personality disorder. What symptom of this diagnosis should the nurse expect to assess, and at what risk is this client for acquiring schizophrenia? 1. Delusions and hallucinations—high risk. 2. Limited range of emotional experience and expression—high risk. 3. Indifferent to social relationships—low risk. 4. Loner who appears cold and aloof—low risk.

2. Individuals diagnosed with schizoid personality disorder are indifferent to social relationships and have a very limited range of emotional experience and expression. They do not enjoy close relationships and prefer to be loners. They appear cold and aloof. Not all individuals who demonstrate the characteristics of schizoid personality disorder progress to schizophrenia, but most individuals diagnosed with schizophrenia show evidence of the characteristics of schizoid personality disorder premorbidly, putting them at high risk for schizophrenia.

A 43 year old man with chronic alcohol use disorder is admitted to the hospital after surgery. After a period of time he becomes confused and paranoid. He is now seeing "small men" in his room. His vital signs become unstable. How many days since he has been admitted to the hospital and what is he experiencing?

3 days, delirium tremens (DT) Delirium tremens: confusion, tachycardia/altered vital signs

The nurse is reviewing lab results for a client diagnosed with a thought disorder who is taking clozapine (Clozaril) 25 mg QD. The following values are documented: RBC4.7 million/mcL, WBC 2000/mcL, and TSH 1.3 mc-IU. Which would the nurse expect the physician to order based on these values? 1. "Levothyroxine sodium (Synthroid) 150 mcg QD." 2. "Ferrous sulfate (Feosol) 100 mg tid." 3. "Discontinue clozapine (Clozaril)." 4. "Discontinue clozapine (Clozaril) and start levothyroxine sodium (Synthroid) 150 mcg QD."

3. A normal adult value of white blood cell (WBC) count is 4500 to 10,000/mcL. This client's WBC count is 2000/mcL, indicating agranulocytosis, which is a potentially fatal blood disorder. There is a significant risk for agranulocytosis with clozapine (Clozaril) therapy. The nurse would expect the physician to discontinue clozapine (Clozaril).

A client on an in-patient psychiatric unit refuses to take medications because, "The pill has a special code written on it that will make it poisonous." What kind of delusion is this client experiencing? 1. An erotomanic delusion. 2. A grandiose delusion. 3. A persecutory delusion. 4. A somatic delusion

3. A persecutory delusion is a type of delusion in which the individual believes he or she is being malevolently treated in some way. Frequent themes include being conspired against, cheated, spied on, followed, poisoned or drugged, maliciously maligned, harassed, or obstructed in the pursuit of long-term goals. The situation described in the question reflects this type of delusion.

The nurse documents that a client diagnosed with schizophrenia is expressing a flat affect. What is an example of this symptom? 1. The client laughs when told of the death of the client's mother. 2. The client sits alone and does not interact with others. 3. The client exhibits no emotional expression. 4. The client experiences no emotional feelings.

3. Flat affect is described as affect devoid of emotional tone. Having no emotional expression is an indication of flat affect.

A nurse is admitting a client to the in-patient unit who is exhibiting bizarre delusions, auditory hallucinations, and incoherent speech. The client is experiencing suicidal ideations and rates mood at 2/10. Based on this clinical picture, the client is manifesting symptoms in what diagnostic category? 1. Paranoid schizophrenia. 2. Brief psychotic disorder. 3. Schizoaffective disorder. 4. Schizophreniform disorder.

3. Schizoaffective disorder is manifested by schizophrenic behaviors with a strong element of symptoms associated with the mood disorders (mania or depression). The client may appear depressed with suicidal ideations. When the mood disorder has been assessed, the decisive factor in the diagnosis is the presence of characteristic schizophrenia symptoms, such as bizarre delusions, prominent hallucinations, or incoherent speech.

The nurse is interviewing a client who states, "The dentist put a filling in my tooth; I now receive transmissions that control what I think and do." The nurse accurately documents this symptom with which charting entry? 1. "Client is experiencing a delusion of persecution." 2. "Client is experiencing a delusion of grandeur." 3. "Client is experiencing a somatic delusion." 4. "Client is experiencing a delusion of influence."

4. A delusion of influence or control occurs when a client believes certain objects or persons have control over his or her behavior. The statement of the client is reflective of a delusion of influence.

A client who is hearing and seeing things others do not is brought to the emergency department. Lab values indicate a sodium level of 160 mEq/L. Which nursing diagnosis would take priority? 1. Altered thought processes R/T low blood sodium levels. 2. Altered communication processes R/T altered thought processes. 3. Risk for impaired tissue integrity R/T dry oral mucous membranes. 4. Imbalanced fluid volume R/T increased serum sodium levels.

4. All physiological problems must be corrected before evaluating thought disorders. In this situation, the psychotic symptoms may be related to the critically high sodium level. If the cause is physiological in nature, the nurse's priority is to assist in correcting the physiological problem. If the client's fluid volume imbalance is corrected, the psychotic symptoms, which are due to the medical condition of hypernatremia, would be eliminated, resulting in an improvement in thought process symptoms. This would improve the client's ability to communicate effectively and decrease the risk of dry mucous membranes.

The client has a long history of schizophrenia, which has been controlled by haloperidol (Haldol). During an admission assessment resulting from an exacerbation of the disease, the nurse notes continuous restlessness and fidgeting. Which medication would the nurse expect the physician to prescribe for this client? 1. Haloperidol (Haldol). 2. Fluphenazine decanoate (Prolixin Decanoate). 3. Clozapine (Clozaril). 4. Benztropine mesylate (Cogentin).

4. Benztropine mesylate (Cogentin) is an anticholinergic medication used for the treatment of extrapyramidal symptoms such as akathisia. The nurse would expect the physician to prescribe this drug for the client's symptoms of restlessness and fidgeting.

A 21-year-old client, being treated for asthma with steroid medication, has been experiencing delusions of persecution and disorganized thinking for the past 6 months. Which factor may rule out a diagnosis of schizophrenia? 1. The client has experienced signs and symptoms for only 6 months. 2. The client must hear voices to be diagnosed with schizophrenia. 3. The client's age is not typical for this diagnosis. 4. The client is receiving medication that could lead to thought disturbances.

4. Steroid medications could precipitate the thought disorders experienced by the client and potentially rule out the diagnosis of schizophrenia. According to the DSM-IVTR criteria for this diagnosis, the thought disturbance cannot be due to the direct physiological effects of a substance.

A nurse is assessing a client in the mental health clinic 6 months after the client's discharge from in-patient psychiatric treatment for schizophrenia. The client has no active symptoms, but has a flat affect and has recently been placed on disability. What should the nurse document? 1. "The client is experiencing symptoms of the schizoid personality phase of the development of schizophrenia." 2. "The client is experiencing symptoms of the prodromal phase of the development of schizophrenia." 3. "The client is experiencing symptoms of schizophrenia." 4. "The client is experiencing symptoms of the residual phase of the development of schizophrenia."

4. Symptoms during the residual phase are similar to those of the prodromal phase, with flat affect and impairment in role function being prominent. This client has recently experienced an active phase of schizophrenia and has been placed on disability, indicating problems with role functioning. The nurse would recognize the symptoms presented as an indication that the client is in the residual phase of schizophrenia.

The nurse is caring for a client with schizophrenia. Orders from the physician include 100 mg chlorpromazine IM STAT and then 50 mg PO bid; 2 mg benztropine PO bid prn. Because benztropine was ordered on a prn basis, which of the following assessments by the nurse would convey a need for this medication? 1. The client's level of agitation increases. 2. The client complains of a sore throat. 3. The client's skin has a yellowish cast. 4. The client develops muscle spasms.

4. The client develops muscle spasms.

Anosognosia

A patient's inability to realize that he or she is ill - caused by the illness itself

Substrate

A substrate is a drug that is metabolized by an CYP enzyme system.

What direct-acting dopamine receptor agonist is recommended to be used in the treatments of neuroleptic malignant syndrome (NMS) fo help lower the dopamine blockade? A) benzotropine (Cogentin) B) bromocriptine (Parlodel) C) dantrolene (Dantrium) D) trihexyphenidyl (Artane)

A) Bromocriptine (Parlodel) is the recommended direct acting dopamine receptor agonist to help decrease the dopamine blockade. Danrolene (Dantrium) is a muscle relaxant. Benzotropine (Cogentin) and Trihexyphenidyl (Artane) are anticholinergic medications used for extrapyramidal side effects (EPS).

Which cytochrome (CYP) enzyme is implicated as a tobacco inducer when an individual is treated on clozapine? A) 1A2 B) 2D6 C) 2C9 D) 2C19

A) 1A2 When an individual is treated on clozapine and decreases tobacco use, the clozapine level with increase, as tobacco is an inducer to the clozapine, and the patient no longer needs the higher dose of clozapine.

Which serotonin receptor antagonism makes an antipsychotic "atypical" A) 5HT2A B) 5HT1A C) 5HT3A D) 5HT4A

A) 5HT2A The mechanism of action that makes an antipsychotic medication "atypical" is related to the 5HT2A receptor antagonism and D2 receptor antagonism.

A patient being treated for major depressive disorder and on sertraline (Zoloft), 150 mg po daily for the past 16 years, presents to the psychiatric mental health practitioner for an outpatient follow-up visit. During the visit she states that she has not been feeling well, reporting the flu. She also states she has not taken her medication in the last five days. Which of the following symptoms would she be describing if you suspect selective serotonin reputable inhibitors (SSRIs) discontinuation syndrome? A) Agitation, nausea, dysphoria, and diequilibrium B) Agitation, nausea, tremor, and ataxia. C) Restlessness, tremor, fever, and shivering. D) Restlessness, headache, increased heart rate, and diarrhea.

A) Agitation; nausea, dysphoria, and disequilibrium The patient has SSRI discontinuation syndrome and would be presenting with flu-like symptoms. If the patient had serotonin syndrome, she would present with symptoms of autonomic instability.

Mr. Jones, a 78 year old patient, presents to the PMHNP for a follow-up medication appointment. Mr. Jones' depression has been successfully treated with citalopram 20 mg by mouth daily. During the visit, Mr Jones complains that in the last 2 or 3 weeks, he has had nausea, fatigue, feeling weak, with a headache and decreased appetite. Which action would be most appropriate for Mr. Jones? A) Assess for other symptoms of hyponatremia and check a serum sodium level. B) Discuss SSRI discontinuation syndrome and stress importance of medication adherence. C) Discuss rates of relapse on a SSRI and increase his medication to citalopram 40 mg by mouth daily. D) Discuss rates of relapse on a SSRI and recommend switching his medication to a different class.

A) Assess for other symptoms of hyponatremia and check serum sodium level. Hyponatremia can occur on SSRIs, and is more common in elderly patients. This patient is reporting symptoms of moderate hyponatremia and needs to be assessed, and his sodium levels need to be checked. Normal sodium level: 135-145 mEq/L

Which of the following statements reflect the current understanding of dopamine (DA) pathways and clinical symptoms in schizophrenia? A) Negative symptoms are related to DA deficit in the cerebral cortex; positive symptoms are related to DA excess in the nucleus accumbens and mesolimbic system. B) Negative symptoms are related to DA excess in the cerebral cortex; positive symptoms are related to DA deficit in the nucleus accumbens and mesolimbic system. C) Negative symptoms are related to DA excess in the mesolimbic system; positive symptoms are related to DA deficit in the substantia nigra and ventral tegmental area. Negative symptoms are related to DA deficit in the mesolimbic system; positive symptoms are related to DA excess in the substantia nigra and ventral tegmental area.

A) Negative symptoms are related to DA deficit in the cerebral cortex; positive symptoms are related to DA excess in the nucleus accumbens and mesolimbic system. Negative symptoms & cognitive impairment are thought to be related to hypoactivity of the mesocortical dopiminergic tract, which by its association with the prefrontal cortex and neocortex contributes to motivation, planning, sequencing of behaviors in time, attention, and social behavior. Positive symptoms (hallucination and delusions) are thought to be caused by dopamine hyperactivity in the mesolimbic tract, which regulates emotion. This hyperactivity could result in overactive modulation of nueurotransmission from the nucleus accumbens.

Which strategy is best for clients who hear voices? A. Avoid certain situations. B. Smoke more cigarettes. C. Decrease caffeine use. D. Take more medication.

A. Avoid certain situations.

Which data is most important to obtain before Sam begins the Zyprexa, which is an atypical antipsychotic? A. Baseline weight. B. Orthostatic blood pressure. C. Complete blood count. D. Screening for tardive dyskinesia.

A. Baseline weight.

Which medication with potentially life-threatening side effects should the nurse expect the healthcare provider to prescribe for clients who do not respond to the use of other antipsychotics? A. Clozapine (Clozaril). B. Haloperidol decanoate (Haldol decanoate). C. Fluphenazine decanoate (Prolixin decanoate). D. Perfenazine (Trilafon).

A. Clozapine (Clozaril).

Which side effects would the nurse most likely observe with fluphenazine (Prolixin), a traditional antipsychotic? A. High extrapyramidal effects, low anticholinergic effects. B. High anticholinergic effects and low extrapyramidal effects. C. Risk for agranulocytosis, fever, and elevated blood pressure. D. Blood dyscrasias such as thrombocytopenia.

A. High extrapyramidal effects, low anticholinergic effects.

Schizophrenia DSM 5 Criteria

A. two (or more) of the following, each present during a 1-month period (or less if successfully treated) and at least one of these has to be 1, 2 or 3: 1. delusions 2. hallucinations 3. disorganized speech 4. grossly disorganized or catatonic behavior 5. negative symptoms 1. pre-morbid phase (not taking care of self) 2. morbid phase (late teens, social withdrawal, very dissolved and suspicious/paranoid behavior) 3. schizophrenia phase is the above criteria 4. residual/remission phase

SchizoAFFECTIVE Disorder DSM 5 Criteria

A. an uninterrupted period of illness during which, at some time, there is either a Major Depressive Episode, a Manic Episode, or a Mixed Episode concurrent with symptoms that meet criterion A for Schizophrenia -note: the major depressive episode must include criterion A1: depressed mood B. During the same period of illness, there have been delusions or hallucinations for at least 2 weeks in the absence of prominent C. Symptoms that meet criteria for a mood episode are present for a substantial portion of the total duration of the active and residual periods of the illness. D. The disturbance is not due to the direct physiological effects of a substance (e.g. a drug of abuse, a medication) or a general medical condition specify type: *bipolar type*: if the disturbance includes a Manic or a Mixed Episode (or a Manic of a Mixed Episode and Major Depressive Episodes) *depressive type*: if the disturbance only includes Major Depressive Episodes

13. A patient taking clozapine (Clozaril) says, "I get plenty of vitamin C by drinking 8 ounces of grapefruit juice each morning." Select the nurse's best response. a. "High doses of Vitamin C support the immune system and general good health." b. "Name another juice you would drink, because grapefruit juice can cause a bad reaction while taking clozapine." c. "Grapefruit juice lessens the effectiveness of your medication. You might need higher doses." d. "New research shows papaya juice is a better source of vitamin C than grapefruit juice."

ANS: B Only the correct option provides vital information based on the cytochrome P-450 enzyme system's involvement in drug metabolism. Clozapine metabolism is inhibited by the ingestion of grapefruit juice, making the likelihood of a toxic reaction to the drug more likely, because the drug accumulates in the body.

A homeless individual diagnosed with serious mental illness, anosognosia, and a history of persistent treatment nonadherence is persuaded to come to the day program at a community mental health center. Which intervention should be the team's initial focus? a. Teach appropriate health maintenance and prevention practices. b. Educate the patient about the importance of treatment adherence. c. Help the patient obtain employment in a local sheltered workshop. d. Interact regularly and supportively without trying to change the patient.

ANS: D Given the history of treatment nonadherence and the difficulty achieving other goals until psychiatrically stable and adherent, getting the patient to accept and adhere to treatment is the fundamental goal to address. The intervention most likely to help meet that goal at this stage is developing a trusting relationship with the patient. Interacting regularly, supportively, and without demands is likely to build the necessary trust and relationships that will be the foundation for all other interventions later on. No data here suggest the patient is in crisis, so it is possible to proceed slowly and build this foundation of trust.

A patient diagnosed with schizophrenia has had multiple relapses. The patient usually responds quickly to antipsychotic medication but soon discontinues the medication. Discharge plans include follow-up at the mental health center, group home placement, and a psychosocial day program. Which strategy should apply as the patient transitions from hospital to community? a. Administer a second-generation antipsychotic to help negative symptoms. b. Use a quick-dissolving medication formulation to reduce "cheeking." c. Prescribe a long-acting intramuscular antipsychotic medication. d. Involve the patient in decisions about which medication is best.

ANS: D Persons with schizophrenia are at high risk for treatment nonadherence, so the strategy needs primarily to address that risk. Of the options here, involving the patient in the decision is best because it will build trust and help establish a therapeutic alliance with care providers, an essential foundation to adherence. Intramuscular depot medications can be helpful for promoting adherence if other alternatives have been unsuccessful, but IM medications are painful and may jeopardize the patient's acceptance. All of the other strategies also apply but are secondary to trust and bonding with providers.

8. A realistic outcome of patient teaching about psychotropic medication is that the patient will: a. understand physiologic responses to drug therapy. b. assess effectiveness of prescribed drugs in controlling symptoms. c. describe onset, peak, and duration of action of each drug prescribed. d. state the purpose, dose, and significant side effects of each drug prescribed.

ANS: D The correct response identifies basic information that each patient should have. Because the information is basic, the outcome, as stated, is realistic. The other options are less basic and less attainable.

1. Which of the following patients is not appropriate for management in the primary care setting? A. A 27-year-old female with new onset depressive symptoms and a scheduled psychiatric intake in 6 weeks B. A 27-year-old male with new onset inattentive symptoms and no history of psychiatric evaluation C. A 32-year-old female who is acutely manic and suicidal and currently has no psychiatric provider D. A 32-year-old male who has been psychiatrically stable on lithium for a year

Answer/Rationale: 1. B. Patients with newly identified symptoms who do not present imminent risk should be referred for a mental health evaluation.

10. A patient who is being treated with lithium 600 mg twice daily has a trough lithium level of 0.9 mEq/L. The patient is currently euthymic. The nurse practitioner does which of the following? A. Decreases the medication to prevent toxicity B. Discontinues the medication to prevent toxicity C. Maintains the current dosage because the plasma level is within range D. Maintains the current dosage because the benefit outweighs the risk

Answer/Rationale: 10. C. The recommended therapeutic range of lithium is between 0.5-1.2 mEq/L.

16. A 42-year-old female presents for follow-up after two emergency room visits for shortness of breath, heart palpitations, sweating, and fear of dying. A comprehensive workup shows no cause of medical illness. A urine drug screen prior to the office visit is positive for marijuana. The patient states, "Ativan really helped in the emergency room. Will you continue it?" What does the FNP advise? A. "Ativan is a safe and effective treatment for your panic attacks." B. "SSRIs are a better option for the long-term management of your panic disorder." C. "Psychotherapy can help you cope with feelings of panic." D. "Marijuana may be causing your anxiety or making it worse. Let's talk more about your use."

Answer/Rationale: 16. D. Substances, such as marijuana, may cause symptoms of anxiety and panic. A timeline of psychiatric symptoms and substance use is needed to differentiate psychiatric disorders from substance-induced symptoms.

18. A family nurse practitioner prescribes Imitrex to a 39-year-old female patient with a history of migraines who is also taking Cymbalta 60 mg daily for anxiety. After taking the first dosage of Imitrex, the patient returns for a same-day visit and reports that she is shivering, has diarrhea, and muscle aches. She is afebrile and her heart rate is normal. The nurse practitioner: A. Suspects a panic attack and advises the patient to continue her medications. B. Suspects Serotonin Syndrome and discontinues the Imitrex. C. Suspects Serotonin Syndrome and discontinues the Cymbalta. D. Advises the patient to go to the emergency department for cardiovascular assessment.

Answer/Rationale: 18. B. Serotonin Syndrome is a serious adverse drug reaction caused by medications with direct and indirect serotonergic effects. Mild cases are typically self-limiting and respond to discontinuation of the offending agent. However, in the case of severe symptoms, including fever, irregular heart rate, and/or seizures, the patient may require hospitalization and supportive care to prevent life-threatening complications.

28. The FNP is seeing a 24-year-old patient who is requesting another prescription for hydrocodone since she lost the previous one. The FNP is concerned about potential abuse. Which of the following is an important first step in the intervention? A. Request that the patient complete a urine drug screen to test levels of opioid in the urine. Inform her that she cannot receive another prescription today. B. Explain to the patient that this is a highly addictive drug and ask if she believes she is beginning to struggle with a dependence on hydrocodone. C. Discontinue the medication and offer her an alternative therapy without abuse potential. D. Complete the CRAFFT questionnaire.

Answer/Rationale: 28. B. The therapeutic alliance is important, and exploring the issue in an honest, non-threatening context is the most effective approach. While a urine toxicology screen and follow-up regarding other potential problems with substance abuse or misuse is important, the first step is exploring the situational context with the patient.

33. A patient treated with clozapine would need to be aware of which fact? A. Clozapine requires laboratory monitoring in the beginning of treatment because of the potential risk of agranulocytosis. B. Clozapine requires blood work monthly for the potential risk of agranulocytosis. C. Clozapine requires blood monitoring if the patient has a family history significant for heart disease due to the potential risk of myocarditis. D. Clozapine requires blood monitoring weekly during the first 6 months of treatment, biweekly for the following 6 months, and then once a month for the entire duration of therapy.

Answer/Rationale: 33. D. This is the FDA requirement for patients prescribed clozapine.

35. Which of the following patients presenting with psychosis would warrant further screening for schizophrenia? A. A 19-year-old male presenting with grossly disorganized speech, flat affect, and who appears suddenly suspicious of family and friends. B. A 20-year-old male with a history of severe mood disturbances, presenting with rapid speech and grandiosity, telling the office personnel that he has been appointed to solve the country's immigration problem. C. A 44-year-old female who is suspicious that her coworkers are trying to sabotage her employment after discovering a hand-written note in her employer's office. D. A 35-year-old male with treatment-resistant depression who has begun to experience auditory hallucinations.

Answer/Rationale: 35. A. This patient is presenting with both positive and negative symptoms typical of schizophrenia. Choice B more accurately reflects a patient experiencing a manic episode of bipolar. Choice C requires further investigation but does not necessarily reflect paranoid ideation. Choice D requires further follow-up, but because psychosis is a symptom of severe depression, depression with psychotic features is more likely.

39. A 30-year-old female reports that she enjoys a "couple of drinks" when she goes out with friends. This occurs 3 or 4 times a week. She states that she has missed work a few times due to hangovers and her girlfriend ended their relationship because she didn't like her "partying so much." What is the most appropriate action by the nurse practitioner? A. Explain to her that this behavior is considered at-risk drinking because she is above the recommended limit for alcohol intake for females. B. Realize that she has to recognize harmful drinking on her own. C. Ask her if she believes the drinking is getting in the way of her job and relationships. Explain to her what harmful drinking means. Perform the CRAFFT and CAGE questionnaire. Screen for other potential substance abuse. D. Refer her for substance abuse treatment.

Answer/Rationale: 39. C. Assessing where the patient is regarding at-risk substance abuse behavior is important. The first step should be an honest, non-threatening conversation, and then an assessment to determine the risk of substance abuse or misuse. While A and D are reasonable choices, the first step is building the therapeutic alliance and assessment. Choice B is not correct because the patient has opened a dialogue regarding the effects of her drinking, and ignoring this would be inappropriate and potentially harmful.

40. A patient presents for his follow-up appointment. He has a stable job as a software programmer, is well groomed with proper hygiene with no symptoms of psychosis. The patient reports that he is suspicious that the office staff has been reading his medical chart. You ask him why he believes this and he responds with, "The HIPAA Confidentiality stuff is just a smoke screen for the government to know our business." When the nurse practitioner attempts to refute this, he becomes angry and hostile. The nurse practitioner would screen for which of the following personality disorders? A. Paranoid Personality Disorder B. Social Anxiety Disorder C. Schizophrenia D. Schizoid Personality Disorder

Answer/Rationale: 40. A. This patient is presenting with signs of paranoid personality disorder, including suspicion, anger, and hostility.

44. A 13-year-old female presents to the NP's office and reports she has been "hearing voices." The NP screens for psychosis. Which of the following descriptions of the "voice" would most concern the treating nurse practitioner? A. "It's like a voice telling me that I am ugly and stupid." B. "I hear it a lot. It sounds like a whisper, but then it tells me that I should hurt people." C. "I only hear the voice when I'm really sad but it sounds like my mom's voice trying to cheer me up." D. "It's the "inner voice" that helps me do the right thing."

Answer/Rationale: 44. B. Auditory hallucinations that are commanding in nature and involve voices instructing the person to harm oneself or another are most concerning and require further assessment and management. Choice A is also concerning and warrants further assessment, but command hallucinations with dangerous content are most concerning. Choices C and D are likely not auditory hallucinations.

49. A patient presents with confusion and is complaining of "muscles feeling weird." Which of the following is the most likely cause? A. Patient recently started on valproic acid. B. Patient recently increased dosage of haloperidol. C. Patient recently discontinued lithium. D. Patient recently began methylphenidate for ADHD.

Answer/Rationale: 49. B. Neuroleptic Malignant Syndrome (NMS) is characterized by confusion, fever, muscle rigidity, and autonomic instability. This is a dangerous potential side effect of haloperidol. Neither valproic acid, lithium, nor adderall are known to cause NMS.

5. A 37-year-old female patient requests a prescription for antidepressant medication. She reports low energy, poor self-esteem, overeating behaviors, difficulty making decisions, and feeling hopeless. When asked how long her symptoms have been occurring, she states, "I've always been this way." The nurse practitioner suspects which of the following? A. Borderline Personality Disorder B. Major Depressive Disorder C. Persistent Depressive Disorder D. Bipolar Disorder

Answer/Rationale: 5. C. The symptoms of persistent depressive disorder occur for at least two years, with no more than 2 months without symptoms.

50. Which of the following classes of medications would the nurse practitioner avoid if a patient makes the following statement: "I'll take the pills as long as it doesn't interfere with my life too much. I'm not great at following rules." A. SNRI B. SSRI C. Second-generation antipsychotics D. MAOI

Answer/Rationale: 50. D. MAOIs require that patients avoid foods high in tyramine. This includes some cheeses, meats, and soy-based products. Failure to avoid these guidelines can result in a hypertensive crisis.

6. A family nurse practitioner is seeing a 33-year-old patient for a follow-up visit for her depression. The patient began sertraline (Zoloft) 50 mg four weeks ago and reports a partial response. The nurse practitioner does which of the following? A. Increases the dosage B. Advises switching to a different medication C. Considers augmenting with another agent D. Refers to a psychiatric provider

Answer/Rationale: 6. A. The next step would be to increase the dosage until full response.

All of the following endocrine findings are suggestive of alcohol dependency, except? A. Splenomegaly B. Gynecomastia C. Testicular atrophy D. Sexual Dysfunction

Answer: - Splenomegaly This is a gastrointestinal (NOT endocrine finding) Also: GI: hepatomegaly and tenderness in the intestine area

A newly admitted patient suffering from schizophrenia is actively hallucinating and has become agitated, as evidenced by a loud voice, verbal threats to staff, and pacing, and clenched fists. What is the nurse's best next step?

Attempt to verbally deescalate the patient and offer PRN meds.

In managing the maintenance phase for Bipolar I disorder, which of the following statements is not supported by current evidence in the literature? A) Both lamotrigine and lithium are superior to placebo in delaying onset of mood-related disorders. B) Extended release formulation of carbamazepine is equivalent to lithium in preventing a manic, hypomanic, or mixed episode. C) Lamotrigine, but not lithium is superior to placebo in preventing a depressive episode. D) Lithium. but not lamotrigine, is superior to placebo in preventing a hypomanic, or mixed episode.

B) Extended release formulation of carbamazepine is equivalent to lithium in preventing a manic, hypomanic, or mixed episode. Extended-release formulation of carbamazepine has demonstrated preliminary efficacy in the tx of acute manic or mixed episodes, but is not currently indicated in maintenance phase tx for Bipolar I disorder. Side effects include dizziness, somnolence, nausea, vomiting, ataxia, blurred vision, dyspepsia, dry mouth, pruritus, and speech disorder. The other responses are supported by current evidence in literature.

The primary goal in working with an actively psychotic, suspicious patient would be to: A) promote interaction with others. B) decrease his/her anxiety and increase trust. C) improve his/her relationship with his/her parents. D) encourage participation in therapy activities.

B) decrease his/her anxiety and increase trust.

You are a nurse practitioner working at an addiction clinic. Today, a 45-year-old male presents to your clinic with the chief complaint, "I am about to lose my family; I have to stop drinking." You understand that it is important to fully screen this patient and you decide to use a tool to do so. The most common tool used in screening for alcohol abuse in this particular patient age group and setting is: A. AUDIT B. CAGE C. CADE D. CRAFFT

B. CAGE C: Need to cut down on your drinking? A: People been annoyed by you mentioning drinking? G: Bad/or guilty about drinking? E: Have you had a drink the first thing in the morning to steady your nerves? Others: AUDIT: Alcohol Use disorders identification test S-MAST: Short Michigan Alcoholism Screening test CRAFFT: Children and adolescents under 21 years of age COWS: Clinical Opiate Withdrawal Scale

How should the nurse interpret Sam's belief that he is a famous movie star and that a limousine driver will arrive to get him later in the day? A. Psychotic thinking. B. Delusional thoughts. C. Flight of ideas. D. Confabulation.

B. Delusional thoughts.

What is the most accurate assessment if the client believes that the healthcare providers are FBI agents and that there are cameras in his apartment to monitor his moves? A. Hallucinations. B. Delusions. C. Confabulation. D. Thought broadcasting.

B. Delusions.

What is the most common cause of relapse in a client with schizophrenia? A. Symptom management. B. Medications. C. Lack of community support. D. Health practices.

B. Medications.

Schizophrenia DSM-5 Criteria Continued

B. Social/occupational dysfunction (adults) or failure to meet expected level of achievement (child/adolescents) C. Duration -- continuous signs of disturbance for at least 6 months, which must include at least 1 month active S&S (or less if successfully treated), which may include prodromal or residual periods D. Exclusions: Mood d/o, substance d/o, general medical condition specify severity level of symptom domain -- none > severe

Antiparkinsonian Agent

Benzotropine (Cogentin) -- anticholinergic Diphenhydramine (Benadryl) -- benzodiazepine Lorazepam (Ativan) -- benzodiazepine

Luvox and Fluoxetine

Both Fluvoxamine / Luvox and Fluoxetine / Prozac are inhibitors of 3A4 so it is best to avoid either of these antidepressants if possible, since it would increase Tegretol levels and possibly lead to toxicity. Psychoed re: grapefruit and inhibition of CYP-3A4

A patient with schizophrenia says to the nurse, "A divine voice coming from space is always telling me to go to New York." What is the best response by the nurse in this situation? A) "What else do they tell you?" B) "I don't think what you are saying is true." C) "Even though the voices are real to you, I am unable to hear any voices speaking." D) "Maybe they are real, but don't listen to them and try to avoid them."

C) "Even though the voices are real to you, I am unable to hear any voices speaking."

You are on call for your service. You arrive at the Emergency Dept to find a 22 year old male patient complaining of having trouble with his eyes. His vitals are stable. He is 5 foot 7 inches tall. His weight is 140 lbs, and his labs are all normal range. He states he has been taking aripiprazole (Abilify) for the last 2 months for his bipolar 1 disorder. He had the dose raised 4 days ago to manage a manic episode. He states that one to three times a day for the past 2 days his eyes will roll upwards for 10-15 min and he will hav trouble seeing until they role back down. After examination you decide to: A) Discontinue to aripiprazole and begin the pt on a gradual regimen of Lamotigrine for 1 week. Have him return to his provider for further management of bipolar disorder. B) Continue the aripiproazole and give benzotropine mesylate. C) Discontinue the aripiprazole and give benzotropine mezylate (Congentin) 1 mg IM. D) Begin a cross titration downward of aripiprazole and upwards of haldol.

C) Discontinue the aripiprazole and give benzotropine mezylate (Congentin) 1 mg IM. This is an aripiprazole-induced oculogyric crisis (acute dystonia)

A 43-year-old male is treated for Bipolar 1 on lithium. Which of the following hematologic changes is associated with lithium? A) Agranulocytosis B) Anemia C) Leukocytosis D) Leukopenia

C) Leukocytosis While the mechanism of action is not clear, the use of lithium can raise white blood cell counts and therefore, CBC should be monitored in patients treated on lithium.

Which theorist is associated with object relations and identification? A. Adler B. Jung C. Freud D. Horney

C. - Freud identified the concept of object relations and identification. Adler identified an intolerable sense of inferiority as the motivating force in life. Jung described personalities as extroverted or introverted. Horney rejected the traditional psychoanalytic belief that women felt disadvantaged because of their genital organs.

The nurse understands that an atypical antipsychotic like olanzapine (Zyprexa) requires what period of time to reach a steady state? A. 2 weeks. B. 4 or more weeks. C. 1 week. D. 2 days.

C. 1 week.

What is the best response by the nurse? A. What are you thinking right now? B. Tell me about how you're feeling. C. Are you hearing any voices? D. I notice that you talk to yourself.

C. Are you hearing any voices?

Which nursing assessment accurately describes Sam's lack of energy? A. Apathy. B. Anhedonia. C. Avolition. D. Affective.

C. Avolition.

Which side effect(s) are characteristic of atypical antipsychotics? A. Increased tardive dyskinesia. B. Less incidence of weight gain. C. Fewer extrapyramidal effects. D. More extrapyramidal effects. E. Dry mouth.

C. Fewer extrapyramidal effects. E. Dry mouth.

Which finding depicts negative symptoms of schizophrenia? A. Difficulty sitting still. B. Rapid and disorganized speech. C. Flat affect and social inattentiveness. D. Delusional statements.

C. Flat affect and social inattentiveness.

Which client behavior validates the need for involuntary hospitalization? A. Beliefs about FBI surveillance. B. Diagnosis of schizophrenia. C. Violence towards father. D. Guarded and suspicious.

C. Violence towards father.

Substance use screening tool interpretation is an important part of advanced assessment and of the role of the psychiatric-mental health nurse practitioner. The most common tool used to assess for alcohol abuse is:

CAGE

Substance use screening tool interpretation is an important part of advanced assessment and of the role of the psychiatric-mental health nurse practitioner. The most common tool used to assess for opiate withdrawal is:

COWS

Atypical Antipsychotics

Clozapine (Clozaril) Risperidone (Risperdal) Olanzapine (Zyprexa) Quietiapine (Seroquel) Ziprasidone (Geodon) Abilify (Aripiprazole) more effective at treating the negative S&S -less effect of dopamine D2 receptors and more strongly antagonize 5-HT2 receptors -5-HT antagonism of mesocortical tract may be key to effectiveness SEs: -fewer SESs than typical -unique SE: metabolic syndrome (weight gain, metabolic changes, diabetes, worsening lipid profile) -increased mortality in elderly with dementia-related psychosis (black box warning)

A patient has cocaine dependency, what measures may be prescribed?

Contingency management (positive and negative contingencies) no FDA medications to treat cocaine addiction

Brandon, a patient on the psychiatric unit, has been diagnosed with schizophrenia. He begins to tell the nurse about how the CIA is looking for him and will kill him if they find him. The most appropriate response by the nurse is: A) "That's ridiculous, Brandon. No one is going to hurt you." B) "The CIA isn't interested in people like you, Brandon." C) "Why do you think the CIA wants to kill you?" D) "I know you believe that, Brandon, but it's really hard for me to believe."

D) "I know you believe that, Brandon, but it's really hard for me to believe."

A patient with a diagnosis of schizophrenia has a history of suicidal ideation and suicide attempts. The PMHNP should consider which antipsychotic medication that is the only antipsychotic to reduce the risk of suicide in schizophrenia? A) Abilify (aripriprazole) B) Latuda (lurasidone) C) Invega (iloperidone) D) Clozaril (clozapine)

D) Clozaril (clozapine) is the only known antipsychotic medication that had been shown to reduce the risk of suicide in patients diagnosed with schizophrenia.

The nurse understands that schizophrenia can be differentiated from psychosis by which assessment? A. Disorganized speech. B. Disorganized behavior. C. Auditory hallucinations. D. Negative symptoms.

D. Negative symptoms.

NMS Intervention

D/C antipsychotic immediately monitor vitals, degree of muscle rigidity, I&O, LOC MD may order bromocriptine (Parlodel) or Dantrolene (Dantrium) or bento to counteract supportive care: IV hydration, renal dialysis, fever reduction (cooling blankets, antipyretics, etc.)

Selected Antipsychotic SEs

Extrapyramidal symptoms (EPS) acute dystonic reactions tardive dyskinesia neuroleptic malignant syndrome metabolic syndrome agranulocytosis

True or False: Substances that have pronounced withdrawal symptoms include alcohol, SSRIs, benzodiazepines, and opiates.

False, SSRIs do not have pronounced withdrawal symptoms (but alcohol, benzos, and opiates do)

Two biomarkers NP 90% sensitivity for heavy alcohol use?

GGT and CDT (not really in purple review book)

What are 4 major dimensions of recovery?

Health, Home, Purpose, and Community

You are a nurse practitioner assessing a patient in the emergency department for acute alcohol withdrawal. You decide to perform a CIWA (Clinical Institute Withdrawal Assessment) for alcohol on the patient. A patient in active alcohol withdrawal will present with all of the following symptoms except: A. Nausea and Vomiting B. Tremors C. Heightened level of sensorium D. Agitation

Heightened level of sensorium will have altered sensorium, not heightened sensorium

CYP 2D6 Inhibition

If a TCA(a substrate for 2D6) is given concomitantly w/an inhibitor of 2D6 (paroxetine, fluoxetine) will cause the TCA level to increase. Must monitor TCA plasma levels, or more likely, avoid concomitant use.

Anticholinergic Agents

MOA: work to restore the natural balance of acetylcholine and dopamine in the CNS interactions -additive anticholinergic effects, including potentially fatal paralytic ileus -concurrent use with haloperidol or phenothiazine -- decreased antipsychotic effect and increased anticholinergic SEs -additive CNS effects with CNS depressants monitor client for these SEs: anticholinergic effects, nausea, GI upset, sedation, dizziness, exacerbation of psychosis, orthostatic hypotension

How do we interact with a patient with delusions/hallucinations?

Meet them where they are and then present reality. Do NOT challenge them. acknowledge, empathize, distract, redirect ex. "I understand you believe the FBI is coming to get you. That must be scary to believe that but you are safe here on the unit. Let's take a walk."

Where is dopamine firing occurring?

Nucleus accumbens and ventral tegmental

What pattern does schizophrenia follow?

Phase 1: premorbid Phase 2: prodromal Phase 3: active psychotic Phase 4: Residual

Substance use screening tool interpretation is an important part of advanced assessment and of the role of the psychiatric-mental health nurse practitioner. The most common tool used to assess for alcoholism in geriatric populations is:

S-MAST Michigan Alcoholism Screening test for Geriatrics

Metabolic Syndrome

SE of certain atypical antipsychotics (e.g. olanzepine (Zyprexa)) patients with schizophrenia are also at greater risk for obesity than other individuals d/t lifestyle and environmental factors

Applicability to Nursing: Psychodynamic Theories - Renewed interest in psychodynamic treatment

Strong emphasis on: - Affect & emotional expression - Examination of topics client avoids - Recurring patterns of behavior, feelings, relationships - The past and influence on the present - Interpersonal relationships - Exploration of wishes, dreams, & fantasies

Brief psychotic disorder

Sudden onset of symptoms that may or may not be preceded by a sever psychological stressor. - symptoms last at least *1 day but less than 1 month* - s/s include emotional turmoil, overwhelming perplexity/confusion, impaired reality: incoherent speech, delusions, hallucinations, bizarre behavior, disorientation

b. buprenorphine plus naloxone

an alternative to methadone that can be used to curb opioid withdrawal sxs is the use of: a. gabapentin b. buprenorphine plus naloxone c. methylnatrexone d. topiramate

d. he should be taking the medication with food

a pt has been taking fluoxetine for 1 wk and c/o mild nausea and diarrhea. you advise that: a. this is a common, long lasting side effect of SSRIs b. he should dc the med c. another antidepressant should be substituted d. he should be taking the medication with food

The primary care NP prescribes disulfiram to a patient who has stopped drinking but continues to have cravings for alcohol. The NP must counsel the patient to: a- abstain from alcohol completely. b- report a garlic taste in the mouth. c- stop taking the drug after a few months. d- increase the drug dose after several months.

a- abstain from alcohol completely. Patients taking disulfiram who consume alcohol experience an uncomfortable and sometimes life-threatening reaction and may have these symptoms up to 14 days after disulfiram is given. A garlic taste is a minor side effect. Patients may take the drug for years but do not need to increase the dose because they can become more sensitive to its effects.

A mother brings her a college-age son to the primary care NP and asks the NP to talk to him about alcohol use. He reports binge drinking on occasion and drinking only beer on weekends. The NP notes diaphoresis, tachycardia, and an easy startle reflex. The NP should: a- admit him to the hospital for detoxification. b- ask him how much he had to drink last night. c- prescribe lorazepam (Ativan) to help with symptoms. d- suggest that he talk to a counselor about alcohol abuse.

a- admit him to the hospital for detoxification. He is showing signs of alcohol withdrawal and possible delirium tremens and so should be admitted to the hospital. Asking him about drinking and suggesting outpatient counseling would be useful for a less emergent condition. The NP should not prescribe a medication to treat delirium tremens on an outpatient basis.

The primary care NP is preparing to prescribe acamprosate for a patient who is an alcoholic. Before initiating treatment with this medication, the NP should: a- assess renal function. b- obtain liver function tests. c- teach the patient never to take the drug with alcohol. d- tell the patient that this medication is used to treat withdrawal symptoms.

a- assess renal function. This drug should not be given if patients have severe renal impairment. LFTs are indicated if signs of liver toxicity occur. Acamprosate does not cause a disulfiram-like reaction and is not used to treat withdrawal.

A patient who is an alcoholic is seen in the clinic, and the primary care NP admits the patient to the hospital for acute withdrawal. The patient has elevated liver enzymes. The NP should expect the inpatient provider to prescribe: a- lorazepam (Ativan). b- diazepam (Valium). c- acamprosate (Campral). d- chlordiazepoxide (Librium).

a- lorazepam (Ativan). Benzodiazepines are used to treat alcohol withdrawal because they demonstrate cross-tolerance with alcohol. Short-acting benzodiazepines are used in patients with liver damage. Lorazepam is a short-acting benzodiazepine. Acamprosate is used to reduce voluntary intake of alcohol and is not used for withdrawal symptoms.

A patient who has been taking alprazolam [Xanax] to treat generalized anxiety disorder (GAD) reports recently stopping the medication after symptoms have improved but reports having feelings of panic and paranoia. Which initial action by the nurse is correct? a. Ask the patient if the medication was stopped abruptly. b. Instruct the patient to resume taking the alprazolam. c. Notify the provider that the patient is experiencing a relapse. d. Suggest that the patient discuss taking buspirone [Buspar] with the provider.

a. Ask the patient if the medication was stopped abruptly.

During the course of a pediatric well-visit check-up, the parent reports that her 16 year old son has been more isolative lately, showing little interest in bathing, and withdrawing from friends. What is the nurse's next step?

additional information is needed to determine whether the patient is experiencing schizophrenia

Active Psychotic Phase

aka acute schizophrenia - phase 3 characterized by episodes of more pronounces symptoms

Coventional ("Typical") Antipsychotics

antipsychotic = major tranquilizer = neuroleptic typical (1st gen) antipsychotics -Chloropromazine (Thorazine) -Haloperidol (Haldol) -Fluphenazine (Prolixin) -Thioridazine (Mellaril) potency -high potency (Haldol) -medium potency (Stelazine) -low potency (Thorazine)

Negative Symptoms

are characterized by restrictions in range and intensity of emotional expression, communication, body language and interest in normal activities *blunted (or flat) affect*: decreased emotional expressiveness, unresponsive, immobile facial appearance, reduced eye contact and body language *alogia*: reduced speech, responses are detached and speech is not fluid *avolition*: lacking motivation, spontaneity, initiative. sitting for lengthy periods or ceasing to participate in work or daily activities *anhedonia*: lacking pleasure or interest in activities that were once enjoyable *social isolation*

Posturing

associated feature - voluntary assumption of inappropriate or bizarre postures for a *long period of time* Note...catatonia is same thing but for short periods of time

At an annual well-woman examination, the primary care nurse practitioner (NP) asks a patient about alcohol consumption. The woman reports she usually consumes six glasses of wine per week and occasionally will consume three or four glasses at a party. The NP smells alcohol on the woman's breath. The woman says she is hung over today. The NP should: a- order liver function tests (LFTs) and a complete blood count. b- question her further about her nightly alcohol consumption—ask what size her wine glasses are. c- consider her at high risk for alcoholism. d- refer her to treatment for alcohol abuse.

b- question her further about her nightly alcohol consumption—ask what size her wine glasses are. Patients with alcohol on their breath should be assessed for alcohol abuse. The woman describes an amount of drinking that would put her at low risk, but alcoholics often minimize their drinking. A first step would be to get more information about how much she is drinking. The laboratory work may be indicated when the degree of suspicion is confirmed. Once alcoholism is diagnosed, she should be referred for treatment.

Typical Antipyschotics

block D2 receptors (antagonist) effectiveness most likely d/t effects on mesolimbic dopamine systems numerous SEs d/t impact on other dopamine tracts -Nigrostriatal -- movement S&S -Mesocortical -- increased negative S&S, cognitive impairment positive S&S > negative S&S

A patient is brought to the clinic by a spouse because of increased somnolence and disorientation. The spouse tells the primary care NP that the patient has been taking oxycodone for postoperative pain. The NP notes a respiratory rate of 8 to 10 breaths per minute. The NP should: a- activate the emergency medical service (EMS) and administer oxygen. b- administer oral methadone (Dolophine). c- administer intramuscular naltrexone (ReVia) d- administer sublingual buprenorphine (Subutex).

c- administer intramuscular naltrexone (ReVia) The patient shows signs of opiate toxicity. Naltrexone is given to reverse the respiratory depression caused by opiate toxicity. The NP would activate EMS if the patient's symptoms worsen. Methadone is used to assist patients addicted to narcotics to withdraw from the drug. Buprenorphine is used to aid with withdrawal symptoms.

4. Due to the need to self-medicate for anxiety, a patient diagnosed with schizophrenia smokes two packs of cigarettes a day. What unique risk does nicotine pose to this patient's health? a. Lung cancer b. Cardiovascular constriction c. Impaired psychotropic medication therapy d. Increased incidence of lung-reacted disorders

c. Impaired psychotropic medication therapy

SEs of Typical Antipsychotics

common: -anticholinergic effects (dry mouth, constipation) -sedation (histamine, blockade) -orthostatic hypotension (beta-adrenergic blockade) -photosensitivity -extrapyramidal symptoms or EPS (Parkinsonian-like effects) -gynecomastia, galactorrhea (2 degrees to elevated prolactin d/t inhibition of tuberoinfudibular system) -cardiac abnormalities -- arrhythmias, flattened T waves, prolonged QT intervals rare -tardive dyskinesia -blood dyscrasias -neuroleptic malignant syndrome -- life threatening

Brandon, a patient on the psychiatric unit, has been diagnosed with schizophrenia. He begins to tell the nurse about how the CIA is looking for him and will kill him if they find him. Brandon's belief is an example of:

delusion of persecution

Disorganized Speech

derailment -- go off topic and can not stay on it, they go off the tracks tangential -- start talking about one thing and then go off on a tangent, they are never going to answer the question that you asked them initially circumstantial -- answer but then digres loose associations concrete thinking incoherence echolalia -- repetition of words and sounds clang associations -- need to rhyme every word word salad -- a bunch of random words put together neologism -- they make up their own words mutism -- they don't talk

Catatonia Specifier

diagnosis "w/catatonic features" includes three (or more) of the following symptoms: 1. Stupor (no psychomotor activity) 2. Catalepsy (passive induction of a posture held against gravity). 3. Waxy flexibility 4. Mutism (exclude if known aphasia) 5. Negativism (opposition or no response to instructions or external stimuli) 6. Posturing (spontaneous & active maintenance of a posture against gravity). 7. Mannerism (odd, circumstantial caricature of normal actions) 8. Stereotypy (repetitive, abnormally frequent, non-goal-directed movements). 9. Agitation, not influenced by external stimuli. 10. Grimacing. 11. Echolalia 12. Echopraxia

Delusions Intervention

encourage client to share content of delusion with you actively listen for theme or feeling tone respond to theme/feeling tone do not reinforce (agree with) the delusion do not discredit (disagree with) the delusion determine degree of reality testing assist client in recognizing reality vs. delusions gradually limit amount of time spent discussing delusion, while increasing amount of time spent discussing current events and people thought interruption or distraction

Amphetamine withdrawal occurs when a previously dependent person stops amphetamine abuse. Which is a symptom?

fatigue

Onset

gender differences -men: late adolescence to mid-20s -women: late 20s to early 30s abrupt or insidious -majority of individuals manifest gradual development of S&S common symptoms of onset -social withdrawal -unusual behavior or ideas -loss of interest in school or work -deterioration in hygiene and grooming -outbursts of anger

2. Laboratory findings for a patient with an alcohol use disorder indicate increased liver function values and: a decreased mean corpuscular volume (MCV) and normal triglyceride levels.elevated potassium and chloride levels.increased MCV and elevated triglyceride levels.increased urine and serum creatinine levels.

increased MCV and elevated triglyceride levels.

Acute Dystonic Reactions

involuntary muscular movements (spasms) or face, arms, legs and neck; occur suddenly; frightening; painful -Oculogyric crisis and Torticolis -occurs more often in children and young males, and with high potency antipsychotics -IM/IV anticholinergics, antihistamine or benzo (Klonopin) -have respiratory support available

Grossly Disorganized or Catatonic Behavior

lack of goal-directed behavior behavior is incomplete, exaggerated or absent catatonia -- extrem psychomotor agitation or retardation

Extrapyramidal Symptoms (EPS)

nigrostriatal dopamine antagonism pseudoparkinsonism -- tremor, shuffling gait, drooling, ridigidityy -- S&S appear within 1-5 days of Tx -- occur most often in women, elderly and dehydrated clients -- other S&S: akinesia (lack of movement) and akathisia (continuous restlessness and fidgeting) incidence: high potency typical > low potency typical > atypical intervention -decrease dose of antipsychotic -change med to atypical -add antiparkinsonian agent

Hallucination Interventions

observe for signs of hallucinating avoid touching client without warning assess content of hallucination -- safety issues? actively listen for theme or feeling tone respond to theme/feeling tone do not reinforce (agree with) or discredit (overly disagree with) the hallucinations determine degree of reality testing assist client in recognizing reality vs. hallucinations teach client about association between anxiety and hallucinations provide distractions to compete with sensory stimulus (interaction or activity)

Impact of Schizophrenia

one in 100 suffer in U.S. 75% of cases begin between 17-25 y/o more common in men than in women (7:5) lifelong, debilitating illness that can devastate individuals and families more than 75% of taxpayer dollars allocated to mental health people with schizophrenia occupy 25% of all inpatient hospital beds 1/3 to 1/2 of homeless suffer from disorder

Delusions

paranoid/persecutory (think people are poisoning their food) grandiose (I'm running for governor 4 years from now, have speeches ready and all) bizarre (aliens are coming in at night and cutting me open) reference (neutral/unrelated phenomenon relate to them, go through newspaper and think there is a special message in there for them) other forms of delusion thinking: thought withdrawal, thought insertion, thought control (other people have control over their thoughts), thought broadcasting (other people can hear their thoughts) MSE: "thought content" example: George Washington is president, aliens are coming, etc.

4. During a mental status examination, an 18-year-old patient reports feeling bugs crawling under the skin. The psychiatric-mental health nurse practitioner documents this statement as: a developmental disorder, and suspects cocaine use.a hypnagogic hallucination, and suspects schizophrenia.a hypnopompic hallucination, and suspects schizophrenia.a perceptual disturbance, and suspects the use of cocaine.

perceptual disturbance and suspects cocaine

Agranulocytosis

potentially fatal blood disorder: WBC below 3,000 mm3 or granulocyte count below 1,500 mm3 abrupt onset of fever, malaise, ulcerative sore throat, leukopenia high incidence (1-2%) associated with Clozaril (clozapine) usually occurs within first 3 months of therapy intervention: may need reverse isolation and antibiotics prevention: -weekly CBC x 6 months, q 2 weeks thereafter -pharmacies will only dispense 1-2 weeks of med at a time and require proof of monitoring

Tardive Dyskinesia

potentially irreversible stereotyped involuntary movements: tongue protrusion, lip smacking, chewing, blinking, grimacing, choreiform movements of limbs and trunk, foot tapping degree: minimal to incapacitating delayed onset: ~6 months into Tx prevalence: 10-20% Txed for 1 year prevention: use lowest possible dose diagnosis: early detection: Abnormal Involuntary Movement Scale (AIMS) management: stop/switch med if possible, decrease dose

Paranoid Delusion Interventions

promote trust -- use primary nurse contact respect client boundaries slowly and gradually increase involvement in group activities as client can tolerate avoid making client monitoring obvious avoid laughing, whispering, joking within earshot of client provide pre-package, unopened food, if necessary

Neuroleptic Malignant Syndrome (NMS)

rare, potentially fatal dysregulation of thermoregulation system S&S: severe parkinsonian muscle rigidity, hyperpyrexia to 107 F, tachycardia, tachypnea, fluctuations in BP, diaphoresis, rapid deterioration of mental status to stupor and coma, elevated CPK, leukocytosis rapid progression over 24-72 hours onset: within hours of initiation of therapy (but has been known to occur even years into therapy) early on, often mistaken for worsening psychosis 20% mortality if untreated more common in dehydrated patients or with high potency antipsychotics

Antipsychotics Indications

schizophrenic disorders, mania, acute psychotic reactions, mood disorder augmentation or mood disorder with psychotic symptoms, delirium/dementia (typicals only) onset of action: -1 to 6 hours tranquilizing -1 to 2 weeks for antipsychotic effects -6 to 8 weeks to reach full effect

Define psychosis

severe mental condition involving: - disorganization of personality (split from reality) - deterioration in social functioning - loss of contact with reality including delusional thinking + - hallucinations

What are the signs of mild alcohol withdrawal?

sooner, a few hours after the last drink: tremors

An inpatient psychiatric nurse is completing his shift charting. His patient denied experiencing hallucinations, but was observed speaking to himself, rapidly looking around the room, and shouting out to no one in particular. Which of the following is appropriate?

the nurse should document, "patient denies AH/VH, but appears to be responding to internal stimuli."

Hallucinations

types: auditory, visual, tactile, olfactory, gustatory assess: duration, intensity, and frequency level of intensity: comforting, condemning, controlling, conquering command hallucinations MSE: perceptions example: auditory command hallucinations, feeling/smelling/seeing/hearing something observable patient behaviors: -laughing, speaking to oneself -moving lips without making sounds -rapid eye movements -delayed verbal responses, as if preoccupied -anxiety -inability to follow conversation -decreased ability to follow verbal commands documentation: + Auditory Hallucinations (AH)/Visual Hallucinations (VH), + Responding to Internal Stimuli (RIS)

d. AST 80 U/L (0 to 31 U/L, mean corpuscular volume (MCV) 103 fL (80-96)

which of the following is most likely to be noted in a 45 y/o woman with lab evidence of chronic excessive alcohol ingestion? a. alanine aminotransferase (ALT) 202 U/L (0 to 31 U/L, mean corpuscular volume (MCV) 70 fL (80-96) b. aspartate transaminase (AST) 149 U/L (0 to 31 U/L, mean corpuscular volume (MCV) 81 fL (80-96) c. alanine aminotransferase (ALT) 88 U/L (0 to 31 U/L, mean corpuscular volume (MCV) 140 fL (80-96) d. AST 80 U/L (0 to 31 U/L, mean corpuscular volume (MCV) 103 fL (80-96)

b. fluoxetine (prozac)

which of the following meds is most likely to cause sexual dysfunction? a. vilazodone (viibryd) b. fluoxetine (prozac) c. nortiptyline (pamelor) d. bupropion (wellbutrin)

b. the development of chronic obstructive airway disease is often associated with regular use

while counseling an adolescent about the risks of marijuana use, the NP considers that: a. sxs of physical and psychological dependency are rarely reported by regular users b. the development of chronic obstructive airway disease is often associated with regular use c. use on a daily basis among teens is significantly less common than that of alcohol d. driving ability is minimally impaired with its use

waxy flexibility

•associated feature - Pt allows body parts to be placed in bizarre or uncomfortable positions - Associated w/ catatonia ex. nurse places arm in upright position to examine. patient does not move arm when nurse says she is done

Inhibitor

Drug that decreases CYP activity and can cause a decrease in the metabolism of another drug (substrates), thus causing an accumulation, and generally leading to an increased (possibly toxic) drug effect.

Inducer

Drug that increased CYP activity and increased metabolism of another drug (substrate), leading to an lower (subtherapeutic) level of the drug (substrate).

A nurse is preparing a patient who will stop taking lorazepam [Ativan] for anxiety and begin taking buspirone [Buspar]. Which statement by the patient indicates a need for further teaching? a. "I can drink alcohol when taking Buspar, but not grapefruit juice." b. "I may need to use a sedative medication if I experience insomnia." c. "I may not feel the effects of Buspar for a few weeks." d. "I should stop taking the Ativan when I start taking the Buspar."

d. "I should stop taking the Ativan when I start taking the Buspar."

d. involved in at least one incarceration r/t substance use in the past six months

DSM-5 criteria for a substance abuse include all of the following EXCEPT: a. desire to cut down and/or has tried unsuccessfully in the past b. inability to maintain major role obligations c. excessive time spent obtaining substance, using substance, or recovering from its effects d. involved in at least one incarceration r/t substance use in the past six months

b. substance overuse resulting in hospitalization

DSM-5 criteria for a substance use d/o include all of the following except: a. substance use in larger amts or over longer period than intended b. substance overuse resulting in hospitalization c. craving or strong desire to use d. substance use in potentially hazardous positions

The most commonly utilized CYP450 isoenzyme for drug metabolism or biotransformation is

3A4

Which patient problem would be suited most to the use of interpersonal therapy? 1 Disturbed sensory perception 2 Impaired social interaction 3 Medication noncompliance 4 Dysfunctional grieving

4 Dysfunctional grieving Interpersonal therapy is considered to be effective in resolving problems of grief, role disputes, role transition, and interpersonal deficit.

CYP 1A2 Inhibition

Several antipsychotics and antidepressants are substrates (substance on which an enzyme acts) for CYP 1A2. Thus in the presence of 1A2 inhibitor (slows down or prevents a particular reaction)- eg Fluvoxamine- their levels will rise. Lowering the dosage of the substrate often must happen in order to avoid side effects.

*C:* Grapefruit juice should not be ingested by persons taking benzodiazepines, since doing so influences the effectiveness of this classification of medications. There are no known interactions between dairy products, leafy green vegetables, or aspirin with benzodiazepines.

An individual taking benzodiazepines should be taught to avoid: A. dairy products. B. leafy green vegetables. C. grapefruit juice. D. aspirin.

Substrates & Inhibitors of CYP 3A4

Antipsychotic Pimozide, the benzo alprazolam & trizolam, anxiolytic buspirone, HMG-CoA reductase inhibitors (statins)& multiple antipsychotics are all substrates for 3A4. Fluvoxamine, fluoxetine, nefazodone are all moderate inhibitors of 3A4.

A nurse is assessing a group of individuals for psychodynamic therapy. Which type of candidates should the nurse choose? Select all that apply. 1 Patients with mild depression 2 Relatively healthy individuals 3 Patients motivated for change 4 Patients who are psychologically minded 5 Patients with borderline personality disorder

2,3,4 Relatively healthy individuals are the best candidates, as psychodynamic therapy is more concerned about "here-and-now" issues. Psychodynamic therapy requires well-functioning individuals, and patients motivated for change can be suitable. Patients who are psychologically minded are relatively healthy with clear area of difficulty and care suitable for psychodynamic therapy. Patients with mild depression are not suitable for psychodynamic therapy, as it requires relatively healthy individuals. Patients with borderline personality disorder are not suitable for psychodynamic therapy, as it requires relatively healthy individuals.

Psychodynamic Theories

- Psychoanalytic theories - Neo-Freudian models - Humanistic theories

Interpersonal psychotherapy identifies several different problem types. Which psychosocial dilemma represents one of these classic areas? Select all that apply. 1 The inability to accept the death of a child 2 Multiple incidences of spousal infidelity 3 Depression that is a result of a forced retirement 4 Loss of employment caused by anger management issues 5 Ritualistic behaviors resulting from obsessive compulsive tendencies

1,2,3,4 In interpersonal psychotherapy, four types of problem areas have been identified: grief, such as complicated bereavement following the death or loss of a loved one; role disputes resulting in conflicts with significant others, including work relationships; role transition issues caused by problematic change in life status or social or vocational roles such as retirement; and interpersonal deficit resulting in the inability to initiate or sustain close relationships. Behaviors resulting from psychiatric illnesses are not one of these problem areas.

A nurse is learning about common cognitive distortions. Which distortions related to drawing conclusions should the nurse evaluate? Select all that apply. 1 Labeling 2 Mind reading 3 Fortune-telling 4 Overgeneralization 5 Emotional reasoning

2,3,5 Mind reading occurs when others' negative thoughts, motives, and responses interfere and the patient jumps to conclusions. Fortune-telling is when a patient anticipates a negative outcome as an established fact and draws conclusion. Emotional reasoning is when a patient draws conclusions based on an emotional state. Labeling is a kind of generalization that leads to forming harsh labels for oneself or others. Overgeneralization is to apply a bad occurrence and believe that it will always be the case.

A nurse is managing a patient suffering from depression. Which short-term therapy should the nurse apply to reduce the patient's psychiatric symptoms? 1 Erikson's ego theory 2 Psychodynamic therapy 3 Interpersonal psychotherapy 4 Freud's psychoanalytical theory

3 Interpersonal psychotherapy is effective short-term therapy that helps in reducing psychiatric symptoms by improving interpersonal relationships, which can be the main cause for depression. Erikson's ego theory gives a developmental model that is more useful for assessment to identify age-appropriate normal skills. Psychodynamic therapy is more suitable for relatively healthy people and is usually considered long-term with a number of sessions. Freud's psychoanalytical theory presents a human developmental process throughout childhood and its relation to human personality.

A nurse is assessing a young boy who is unwilling to drive on the road following a minor road accident. Which cognitive distortion is applicable to this patient? 1 Labeling 2 Mental filter 3 Overgeneralization 4 All-or-nothing thinking

3 Overgeneralization

A depressed patient tells the nurse "I have never been able to have a successful relationship. I am not able to trust anybody. Is there a problem with me?" This type of behavior suggests an issue in what area? 1 Grief 2 Role disputes 3 Role transitions 4 Interpersonal deficit

4 Interpersonal deficit According to interpersonal psychotherapy, there are four types of problems. They are grief, role disputes, role transition, and interpersonal deficit. Interpersonal deficit refers to the problems where an individual is unable to initiate or sustain close relationships. In this case, the patient has problems with developing trust and having close relationships. This is an example of interpersonal deficit. Grief refers to the problems occurring during the process of bereavement following death or loss of a loved one. Role disputes refer to the problems arising due to conflicts with an important person in an individual's life. Role transitions refer to the problematic situations due to changes in life status, social, and vocational role.

20. Consider this comment from a therapist: "The patient is homosexual but has kept this preference secret. Severe anxiety and depression occur when the patient anticipates family reactions to this sexual orientation." Which perspective is evident in the speaker? a. Theory of interpersonal relationships c. Psychosexual theory b. Classical conditioning theory d. Behaviorism theory

A

33. Which patient is the best candidate for brief psychodynamic therapy? a. An accountant with a loving family and successful career who was involved in a short extramarital affair b. An adult with a long history of major depression who was charged with driving under the influence (DUI) c. A woman with a history of borderline personality disorder who recently cut both wrists d. An adult male recently diagnosed with anorexia nervosa

A

The PMH-RN utilizes psychodynamic treatment with clients seeking help for depression. Which areas should the nurse prioritize in the assessment? (Select all that apply) A.Examine topics the client avoids B.Determine unmet needs of the client C.Explore dreams of the client D.Discover if anxiety is repressing desires E.Relationships with others

A, C, E - Psychodynamic therapists will examine topics clients will try to avoid, explore the dreams of the clients and determine the client's relationships with others. Maslow's hierarchy of needs is utilized when determining if the needs of the client are or have been met. Gestalt therapy will assist the PMH-RN in determining the amount of anxiety which may be contributing to repression of the client's desires.

After taking chlorpromazine (Thorazine) for 1 month, a client presents to an ED with severe muscle rigidity, tachycardia, and a temperature of 105oF (40.5C). Which medical diagnosis and treatment should a nurse anticipate when planning care for this client? 1. Neuroleptic malignant syndrome treated by discontinuing Thorazine and administering dantrolene (Dantrium) 2. Neuroleptic malignant syndrome treated by increasing Thorazine dosage and administering an antianxiety medication 3. Dystonia treated by administering trihexyphenidyl (Artane) 4. Dystonia treated by administering bromocriptine (Parlodel)

ANS: 1 Rationale: The nurse should expect that an ED physician would diagnose the client with neuroleptic malignant syndrome and treat the client by discontinuing chlorpromazine and administering dantrolene. Neuroleptic malignant syndrome is a potentially fatal condition characterized by rigidity, fever, altered consciousness, and autonomic instability. The use of typical antipsychotics is largely being replaced by atypical antipsychotics because they ha

A client diagnosed with schizophrenia spectrum disorder is prescribed clozapine (Clozaril). Which client symptoms, related to the side effects of this medication, should prompt a nurse to intervene immediately? 1. Sore throat, fever, and malaise 2. Akathisia and hypersalivation 3. Akinesia and insomnia 4. Dry mouth and urinary retention

ANS: 1 Rationale: The nurse should intervene immediately if the client experiences signs of an infectious process, such as a sore throat, fever, and malaise, when taking the atypical antipsychotic drug clozapine. Clozapine can have a serious side effect of agranulocytosis, in which a potentially fatal drop in white blood cells can occur, leading to infection.

A nurse is administering risperidone (Risperdal) to a client diagnosed with schizophrenia spectrum disorder. Which of the following client symptoms would most likely decrease because of the therapeutic effect of this medication? (Select all that apply.) 1. Somatic delusions 2. Social isolation 3. Gustatory hallucinations 4. Flat affect 5. Clang associations

ANS: 1, 3, 5 Rationale: The nurse should expect that risperidone would be effective treatment for the positive symptoms of somatic delusions, gustatory hallucinations, and clang associations. Risperidone is an atypical antipsychotic that has been effective in the treatment of the positive symptoms of schizophrenia and in maintenance therapy to prevent exacerbation of schizophrenic symptoms.

After 6 months of taking imipramine (Tofranil) for depressive symptoms, a client complains that the medication doesn't seem as effective as before. Which question should the nurse ask to determine the cause of this problem? 1. "Are you consuming foods high in tyramine?" 2. "How many packs of cigarettes do you smoke daily?" 3. "Do you drink any alcohol?" 4. "Are you taking St. John's wort?"

ANS: 2 Rationale: Imipramine is a tricyclic antidepressant. Smoking should be avoided while receiving tricyclic therapy. Smoking increases the metabolism of tricyclics, requiring an adjustment in dosage to achieve the therapeutic effect. Alcohol potentiates the effects of antidepressants. Tyramine is only an issue when MAOI medications are prescribed. Concomitant use of St. John's wort and SSRIs, not tricyclics, increases, not decreases the effects of the drug.

A nurse prepares to assess a client using the Abnormal Involuntary Movement Scale (AIMS). Which side effect of antipsychotic medications led to the use of this assessment tool? 1. Dystonia 2. Tardive dyskinesia 3. Akinesia 4. Akathisia

ANS: 2 Rationale: The AIMS is a rating scale that was developed in the 1970s by the National Institute of Mental Health to measure involuntary movements associated with tardive dyskinesia.

A nurse is caring for a client who is experiencing a flat affect, paranoid delusions, anhedonia, anergia, neologisms, and echolalia. Which statement correctly differentiates the client's positive and negative symptoms of schizophrenia? 1. Paranoid delusions, anhedonia, and anergia are positive symptoms of schizophrenia. 2. Paranoid delusions, neologisms, and echolalia are positive symptoms of schizophrenia. 3. Paranoid delusions, anergia, and echolalia are negative symptoms of schizophrenia. 4. Paranoid delusions, flat affect, and anhedonia are negative symptoms of schizophrenia.

ANS: 2 Rationale: The nurse should recognize that positive symptoms of schizophrenia include, but are not limited to, paranoid delusions, neologisms, and echolalia. The negative symptoms of schizophrenia include, but are not limited to, flat affect, anhedonia, and anergia. Positive symptoms reflect an excess or distortion of normal functions. Negative symptoms reflect a diminution or loss of normal functions.

An aging client diagnosed with schizophrenia spectrum disorder takes an antipsychotic and a beta-adrenergic blocking agent for hypertension. Understanding the combined side effects of these drugs, which statement by a nurse is most appropriate? 1. "Make sure you concentrate on taking slow, deep, cleansing breaths." 2. "Watch your diet and try to engage in some regular physical activity." 3. "Rise slowly when you change position from lying to sitting or sitting to standing." 4. "Wear sunscreen and try to avoid midday sun exposure."

ANS: 3 Rationale: The most appropriate statement by the nurse is to instruct the client to rise slowly when changing positions. Antipsychotic medications and beta blockers cause a decrease in blood pressure. When given in combination, the additive effect of these drugs places the client at risk for developing orthostatic hypotension.

A client is diagnosed with schizophrenia spectrum disorder. A physician orders haloperidol (Haldol) 50 mg bid, benztropine (Cogentin) 1 mg prn, and zolpidem (Ambien) 10 mg HS. Which client behavior would warrant the nurse to administer benztropine? 1. Tactile hallucinations 2. Tardive dyskinesia 3. Restlessness and muscle rigidity 4. Reports of hearing disturbing voices

ANS: 3 Rationale: The symptom of tactile hallucinations and reports of hearing disturbing voices would be addressed by an antipsychotic medication such as haloperidol. Tardive dyskinesia, a potentially irreversible condition, would warrant the discontinuation of an antipsychotic medication such as haloperidol. An anticholinergic medication such as benztropine would be used to treat the extrapyramidal symptoms of restlessness and muscle rigidity.

A client diagnosed with schizophrenia spectrum disorder takes an antipsychotic agent daily. Which assessment finding should a nurse prioritize? 1. Respirations of 22 beats/minute 2. Weight gain of 8 pounds in 2 months 3. Temperature of 104F (40C) 4. Excessive salivation

ANS: 3 Rationale: When assessing a client diagnosed with schizophrenia spectrum disorder who takes an antipsychotic agent daily, the nurse should immediately address a temperature of 104F (40C). A temperature this high may indicate neuroleptic malignant syndrome, a life-threatening side effect of antipsychotic medications.

Which medication orders should a nurse anticipate for a client who has a history of benzodiazepine withdrawal delirium? 1. Haloperidol (Haldol) and fluoxetine (Prozac) 2. Carbamazepine (Tegretol) and donepezil (Aricept) 3. Disulfiram (Antabuse) and lorazepan (Ativan) 4. Chlordiazepoxide (Librium) and phenytoin (Dilantin)

ANS: 4 Rationale: The nurse should anticipate that a physician would order chlordiazepoxide and phenytoin for a client who has a history of benzodiazepine withdrawal delirium. It is common for long-lasting benzodiazepines to be prescribed for substitution therapy. Phenytoin is an anticonvulsant used to prevent seizures.

A 60-year-old client diagnosed with schizophrenia spectrum disorder presents in an ED with uncontrollable tongue movements, stiff neck, and difficulty swallowing. Which medical diagnosis and treatment should a nurse anticipate when planning care for this client? 1. Neuroleptic malignant syndrome treated by discontinuing antipsychotic medications 2. Agranulocytosis treated by administration of clozapine (Clozaril) 3. Extrapyramidal symptoms treated by administration of benztropine (Cogentin) 4. Tardive dyskinesia treated by discontinuing antipsychotic medications

ANS: 4 Rationale: The nurse should expect that an ED physician would diagnose the client with tardive dyskinesia and discontinue antipsychotic medication. Tardive dyskinesia is a condition of abnormal involuntary movements of the mouth, tongue, trunk, and extremities that can be a side effect of typical antipsychotic medications.

6. A patient has taken clonazepam (Klonopin) for years to manage panic attacks but impulsively stopped the drug. Thirty hours later, the patient comes to the emergency room in distress. What is the nurse's priority action? a. Begin seizure precautions. b. Refer the patient for addiction counseling. c. Institute a behavior modification program. d. Prepare to administer flumazenil (Romazicon)

ANS: A There is evidence to suggest that abrupt withdrawal of clonazepam might precipitate status epilepticus. With this in mind, withdrawal from long-term use warrants seizure precautions. The patient does not have an overdose, so flumazenil (Romazicon) is not indicated. The other options are inappropriate.

20. A health care provider prescribes lorazepam (Ativan) for an anxious older adult. What is the nurse's best action? a.Assess for a history of drug abuse. b.Administer the drug as prescribed. c.Confer with the health care provider. d.Assess the patient's pupillary reaction to light.

ANS: B Lorazepam is a benzodiazepine that has a short half-life. It might be administered safely to older adult patients, although the dose should often be modified downward. It is inadvisable to give benzodiazepines with longer half-lives to older adult patients.

A patient started diazepam (Valium) 5 mg b.i.d. 6 months ago. Now, the patient requires 10 mg to achieve the same effect. This phenomenon results from: a. addiction. b. tolerance. c. dependence. d. disinhibition.

ANS: B Tolerance is the need for increasing amounts of a substance to achieve the same effects. The other terms, defined in the text, do not account for this phenomenon

3. A patient in the emergency room is suspected to have an overdose of benzodiazepines. Which assessment findings validate this diagnosis? Select all that apply. a. Blood pressure 180/94 b. Diminished reflexes c. Hypervigilance d. Somnolence e. Confusion

ANS: B, D, E Benzodiazepine toxicity may result from an overdose. Assessment findings include hypotension, somnolence, confusion, and diminished reflexes.

7. Which patient has the greatest risk for overdose with a benzodiazepine? A patient who: a. takes the drug with antacids b. takes the drug before meals c. combines the drug with alcohol d. experiences depression as well as anxiety

ANS: C Benzodiazepines taken with alcohol produce marked CNS depression, even death. Antacids prevent absorption. Larger doses of benzodiazepines by themselves are rarely lethal. Depression in and of itself is not an indicator of overdose risk. Suicidal ideation might be present, but benzodiazepines by themselves are rarely lethal.

An adult diagnosed with a serious mental illness says, "I do not need help with money management. I have excellent ideas about investments." This patient usually does not have money to buy groceries by the middle of the month. The nurse assesses the patient as demonstrating: a. rationalization. b. identification. c. anosognosia. d. projection.

ANS: C The patient scenario describes anosognosia, the inability to recognize one's deficits due to one's illness. The patient is not projecting an undesirable thought or emotion from himself onto others. He is not justifying his behavior via rationalization and is not identifying with another.

*B:* Abrupt withdrawal can cause troublesome to serious side effects such as agitation, tremor, irritability, insomnia, vomiting, sweating, convulsions, and even psychotic episodes; therefore, gradual tapering is imperative. It is incorrect to say that anxiety can never be resolved without medication. Suggesting other medications does not address the patient's thoughts about discontinuing the diazepam. While it is true that alcohol should not be ingested when taking diazepam, this information is not related to the patient's concerns.

An individual has taken Valium (diazepam) for the past 4 years and is considering discontinuing the medication. What information should the nurse reinforce for the patient? A. "Anxiety seldom gets better without medication." B. "Diazepam should not be discontinued abruptly." C. "Buspirone is a better choice when anxiety also occurs." D. "It is important to remember not to mix alcohol with diazepam."

3. A patient with agoraphobia took alprazolam (Xanax) 0.5 mg t.i.d. for 3 months and then discontinued it. The next day, the patient called the nurse complaining of insomnia, shakiness, and sweating. The nurse's assessment questions should focus on whether the patient: a. may have also been drinking alcohol or taking antihistamines. b. has built up tolerance to alprazolam and needs an increased dose. c. is having withdrawal symptoms from abrupt discontinuation of the drug. d. has progressed to panic attacks and needs a nonbenzodiazepine medication.

ANS: C The patient's symptoms suggest benzodiazepine withdrawal. The nurse knows that patients often attempt to manage their own care by discontinuing medication when they begin to feel better. Benzodiazepines should be slowly withdrawn if withdrawal symptoms are to be avoided. Drinking alcohol would result in different symptoms. Development of tolerance and panic attack symptoms would be different from those mentioned.

19. The half-life of a benzodiazepine drug is 20 hours for a young adult, but for an older adult it is likely to be: a.10 hours. b.30 hours. c.40 hours. d. 80 hours.

ANS: D Because of decreased liver size and function in older adults, the half-life of benzodiazepines is markedly lengthened. Benzodiazepines with long half-lives are unsuitable for older adults.

Lorazepam (Ativan) reduces anxiety by: a. increasing serotonin levels. b. blocking dopamine receptors. c. depressing norepinephrine levels. d. potentiating gamma-aminobutyric acid (GABA).

ANS: D Benzodiazepines enhance the effects of the inhibitory neurotransmitter GABA, slowing neuronal firing. They do not affect dopamine, serotonin, or norepinephrine.

An outpatient diagnosed with schizophrenia attends programming at a community mental health center. The patient says, "I threw away the pills because they keep me from hearing God." Which response by the nurse would most likely to benefit this patient? a. "You need your medicine. Your schizophrenia will get worse without it." b. "Do you want to be hospitalized again? You must take your medication." c. "I would like you to come to the medication education group every Thursday." d. "I noticed that when you take the medicine, you have been able to hold a job you wanted."

ANS: D The patient appears not to understand that he has an illness. He has stopped his medication because it interferes with a symptom that he finds desirable (auditory hallucinations—the voice of God). Connecting medication adherence to one of the patient's goals (the job) can serve to motivate the patient to take the medication and override concerns about losing the hallucinations. Exhorting a patient to take medication because it is needed to control his illness is unlikely to be successful; he does not believe he has an illness. Medication psychoeducation would be appropriate if the cause of nonadherence was a knowledge deficit.

An outpatient diagnosed with schizophrenia tells the nurse, "I am here to save the world. I threw away the pills because they make God go away." The nurse identifies the patient's reason for medication nonadherence as: a. poor alliance with clinicians. b. inadequate discharge planning. c. dislike of medication side effects. d. lack of insight associated with the illness.

ANS: D The patient's nonadherence is most closely related to lack of insight into his illness. The patient believes he is an exalted personage who hears God's voice rather than an individual with a serious mental disorder who needs medication to control his symptoms. While the distracters may play a part in the patient's nonadherence, the correct response is most likely.

Cytochrome P450 (CYP) System

About 60% of psychiatric drugs are metabolized through an CYP450. Many drugs may increase or decrease the activity of various CYP isozymes either by inducing the biosynthesis of a CYP (INDUCER) or by directly inhibiting the activity of the CYP (INHIBITION). This is a major source of adverse drug interactions, since changes in CYP enzyme activity may affect the metabolism and clearance of various drugs.

Which benzo has the shortest half life

Alprazolam

17. The parent of a child diagnosed with schizophrenia tearfully asks the nurse, "What could I have done differently to prevent this illness?" Select the nurse's best response. a. "Although schizophrenia results from impaired family relationships, try not to feel guilty. No one can predict how a child will respond to parental guidance." b. "Schizophrenia is a biological illness resulting from changes in how the brain and nervous system function. You are not to blame for your child's illness." c. "There is still hope. Changing your parenting style can help your child learn to cope effectively with the environment." d. "Most mental illnesses result from genetic inheritance. Your genes are more at fault than your parenting."

B

19. A patient had psychotherapy weekly for 5 months. The therapist used free association, dream analysis, and facilitated transference to help the patient understand conflicts and foster change. Select the term that applies to this method. a. Rational-emotive behavior therapy b. Psychodynamic psychotherapy c. Cognitive-behavioral therapy d. Operant conditioning

B

Which theorist introduced the concept of the nurse-client relationship? A. Orlando B. Peplau C. Parse D. Orem

B. Peplau, in her book, Interpersonal Relations in Nursing, introduced the concept of the nurse-client relationship. Orlando helped nurses to focus on the whole client rather than the disease or institutional demands. Parse views humans as indivisible, unpredictable, ever-changing coauthors, and experts about their lives. Orem's theory focuses on self-care.

Five Cytochrome P450 (CYP) Systems

CYP 1A2 CYP 2D6 CYP 2C9 CYP 2C19 CYP 3A4

CYP 3A4 Induced by Carbamazepine

Carbamazepine (tegretol) anticonvulsant, Tegretol is a potent inducer of hepatic 3A4 and is also known to be an inducer of CYP1A2, 2B6, 2C9/19 and may therefore reduce plasma concentrations of co-medications mainly metabolized by CYP 1A2, 2B6, 2C9/19 and 3A4, through induction of their metabolism. This results in the acceleration of the metabolism of concurrently prescribed anticonvulsants, particularly valproic acid, clonazepam, ethosuximide, lamotrigine, topiramate, tiagabine and remacemide. The metabolism of many other drugs such as tricyclic antidepressants, antipsychotics, steroid oral contraceptives, glucocorticoids, oral anticoagulants, cyclosporin, theophylline, chemotherapeutic agents and cardiovascular drugs can also be induced, leading to a number of clinically relevant drug interactions.

Carbamazepine and Valproate

Carbamazepine when given with valproate causes increased carbamazepine levels and decreased valproate levels.

CYP 1A2 & Smoking

Cigarette smoking, can increase 1A2 and lower drug concentrations of drugs metabolized by this enzyme- such as olanzapine, clozapine, zotepine, etc. Smokers may need much higher doses of these drugs than non-smokers. Smoking is a very strong inducer of 1A2. So levels are decreased, if someone is a heavy smoker any medication metabolized through 1A2 has to be increased. So must uptitrate the dose of the medication due to the high inducer level of smoking. Someone on olanzapine, they are a smoker, and then they stop smoking, [smoking decreases the level of zyprexa], their drug level has gone up. So then you would down-titrate slowly. Exact percentage is not clear but it could be up to 50%.

What are some CYP 450 3A4 inhibitors (medications)

Clarithromycin Intraconazole Azithromycin NOT Erythromycin

22. A person says, "I was the only survivor in a small plane crash. Three business associates died. I got depressed and saw a counselor twice a week for 4 weeks. We talked about my feelings related to being a survivor, and I'm better now." Which type of therapy was used? a. Milieu therapy c. Behavior modification b. Psychoanalysis d. Interpersonal psychotherapy

D

a. giving him time and support during this period of acute grief

J. is a 47 y/o man who reports constant sadness following the death of his wife in a motor vehicle accident 2 weeks ago. he has not been able to function at work and avoids socializing with friends and family. you recommend: a. giving him time and support during this period of acute grief b. weekly psychotherapy sessions c. prescribing an anxiolytic to help with grief sxs d. psychotherapy plus a RX for an antidepressant

c. continued or escalated stress from the troubled marriage

S. is a 34 y/o married woman who is dx with major depressive disorder. she feels tha tis is likely associated with stress resulting from her troubled marriage. she is initiated on an SSRI and reports initial improvement in sxs. however, over the following months, the medication loses its effectiveness despite her insistence that she is being adherent with the dosing regimen. this is likely a result of: a. inadequate dose of the med b. development of tolerance to the SSRI c. continued or escalated stress from the troubled marriage d. missed doses despite her insistence on compliance

a. bothersome but not life-threatening

SSRI withdrawal syndrome is best characterized as: a. bothersome but not life-threatening b. potentially life-threatening c. most often seen with dc of agents with a long half-life d. associated with seizure risk

*D:* Antacids impair the absorption rate of benzodiazepines, thus decreasing their effectiveness. Assessing the possible effect the antacids have had on the patient's anxiety management is the initial focus of the interview. Determining triggers and the details of the indigestion are relevant but are not of initial importance.

The nurse learns that a patient prescribed a benzodiazepine for anxiety has been self-medicating with over-the-counter antacids for recurring indigestion. Which statement will the nurse make to initially assess the patient? A. "Do you believe your antianxiety medicine has been working as effectively?" B. "Could the indigestion be related to your anxiety issues?" C. "When did the indigestion first present as a problem?" D. "How many doses of antacids do you usually take each day?"

Age related changes in the elder include

Total body water decreases Body weight as fat increases Increase in gastric pH NOT Increase in serum albumin

*B:* When a benzodiazepine is withdrawn from the dependent person, symptoms such as agitation, tremor, irritability, insomnia, vomiting, sweating, and even convulsions might be experienced. None of the remaining options is generally associated with benzodiazepine withdrawal.

What is the most serious symptom of benzodiazepine withdrawal? A. Visual hallucinations B. Convulsions C. Cardiac arrest Incorrect D. Respiratory arrest

*D:* Use of benzodiazepines for medical management of detoxification assures a safe, controlled, and predictable withdrawal from alcohol to lessen the occurrence of delirium tremens. The use of benzodiazepines is not meant to substitute another addictive substance for the alcohol. Alcohol detoxification results in serious side effects that are best managed in a controlled, safe medical environment that is prepared to manage the patient's care.

What is the rationale for using benzodiazepines to detoxify patients diagnosed with chronic alcoholism? A. They prepare the patient for using prescribed drugs rather than alcohol. B. They allow for the gradual introduction of a less addictive substance. C. They permit the detoxification process to occur in a hospital setting. D. They significantly help manage the withdrawal symptoms

c. i am concerned about your health and safety

a 42 y/o man who has a long-standing hx of alcohol abuse presents for primary care. he admits to drinking 12 to 16 beers daily for 10 yrs. he states "I really do not feel like the booze is a problem. i get to work every day. your most appropriate response is: a. work is usually the last thing to go in alcohol abuse b. your family has suffered by your drinking c. i am concerned about your health and safety d. participating in a support grp can help you understand why you drink

c. this is a common adverse effect of SSRI that is unlikely to resolve without adjustment to his therapy

a 44 y/o man has been taking an SSRI for the past 4 months and c/o new onset of sexual dysfunction and difficulty achieving orgasm. you advise him that: a. this is a transiet side effect often seen int he first weeks of txment b. switching to another SSRI would likely be helpful c. this is a common adverse effect of SSRI that is unlikely to resolve without adjustment to his therapy d. he should see an urologist for further eval

b. rapid withdrawal in this situation can lead to tremors and hallucinations

a middle-aged woman who has taken tx dose of lorazepam for the past 6 yrs wishes to stop taking the med. you advise her that: a. she can dc the drug immediately if she believes it no longer helps with her sxs b. rapid withdrawal in this situation can lead to tremors and hallucinations c. she should taper down the dose of the med over the next week d. GI upset is typically reported during the first week of benzodiazepine withdrawal

Questions to ask when anticipating grapefruit juice-drug interactions whether the target drug has which charecteristics

a) Metabolized or biotransformed by intestinal CYP 450 3A4 b) Low bioavailability c) Narrow therapeutic index NOT Parenteral route of administration

A nurse is performing an admission assessment on a patient. The patient reports taking alprazolam [Xanax] for "nerves." The nurse knows that this patient is most likely being treated for which condition? a. Generalized anxiety disorder b. Obsessive-compulsive disorder (OCD) c. Panic disorder d. Post-traumatic stress disorder (PTSD)

a. Generalized anxiety disorder

1: agitation 2: tremor 3: irritability 4: vomiting 5: sweating 6: convulsions

abrupt withdrawal of benzodiazepines (6)

A patient reports having occasional periods of tremors, palpitations, nausea, and a sense of fear, which usually dissipate within 30 minutes. To treat this condition, the nurse anticipates the provider will prescribe a drug in which drug class? a. Benzodiazepines b. Monoamine oxidase inhibitors c. Selective serotonin reuptake inhibitors d. Tricyclic antidepressants

c. Selective serotonin reuptake inhibitors

An agitated, extremely anxious patient is brought to the emergency department. The prescriber orders a benzodiazepine. The nurse understands that benzodiazepines are used in this clinical situation based on which principle? a. Benzodiazepines have a very short half-life. b. Physical dependence is not a risk when taking benzodiazepines. c. Benzodiazepines are known to cure generalized anxiety. d. Benzodiazepines have a rapid onset of action.

d. Benzodiazepines have a rapid onset of action.

d. low level depression

dysthymia is characterized by: a. suicidal thoughts b. multiple incidents of harming oneself c. social isolation d. low level depression

b. MDMA (e.g. Molly)

hyperthermia and a racing heart rate is a potentially life-threatening presentation for a person using: a. cannabis b. MDMA (e.g. Molly) c. LSD d. barbiturates

A drug that increases the rate of another drug's metabolism or biotransformation via CYP 450 isoenzyme is known as a(n):

inducer

c. 28 y/o man who occasionally "skips a dose" of the medication

of the following individuals in need of an antidepressant, who is the best candidate for fluoxetine (prozac) txment? a. 80 y/o woman with depressed mood for 1 yr after the death of her husband b. 45 y/o man with mild hepatic dysfunction c. 28 y/o man who occasionally "skips a dose" of the medication d. 44 y/o man with decreased appetite

a. this is a common, transient side effect of SSRIs

one week into sertraline (zoloft) txment, a pt c/o new onset recurrent dull frontal headache that is relieved promptly with acetaminophen. which of the following is true in this situation? a. this is a common, transient side effect of SSRIs b. she should dc the med c. fluoxetine should be substituted d. desipramine should be added

d. products with longer duration of action

risk of benzodiazepine misuse can be minimized by use of: a. agents with a shorter half-life b. the drugs as an "as needed" rescue med for acute anxiety c. more lipophilic products d. products with longer duration of action

d. products with long duration of action

risk of benzodiazepine misuse is minimized by use of: a. agents with a shorter T 1.2 b. the drug as an as-needed rescue med for acute anxiety c. more lipophilic products d. products with long duration of action

A drug that is metabolized by a given CYP 450 isoenzyme is known as a

substrate

What is the anticipated outcome when a CYP 1A2 substrate is taken with a CYP 1A2 inducer?

substrate clearance is enhanced

d. elimination of sadness

successful tx of a pt with reactive depression associated with a loss (e.g. death of a loved one, would expect all of the following results EXCEPT: a. elevated mood b. restored function c. improved decision making ability d. elimination of sadness

a. diminished effect with the same amt of substance used

the DSM5 criteria for substance use tolerance includes: a. diminished effect with the same amt of substance used b. desiring to get an amplified effect with higher doses c. ability to decrease the frequency of substance use d. absence of withdrawal sxs when substance is not used for a prolonged period

b. hypertension

when assessing a person with acute opioid WITHDRAWAL, you expect to find: a. constipation b. hypertension c. hypothermia d. somnolence

c. decreasing the dose by 25% per week

when d/c benzodiazepine txment after prolonged use, you recommend: a. terminating txment immediately b. decreasing the dose by 20% per day c. decreasing the dose by 25% per week d. decreasing the dose by 50% per week

c. chest pain

when providing care for a middle age man with acute cocaine intoxication, you inquire about: a. feelings of anxiety b. difficulty maintaining sleep c. chest pain d. abdominal pain

d. paroxetine

which of the following SSRIs is associated with the greatest anticholinergic effect? a. fluvoxamine b. sertraline c. fluoxetine d. paroxetine

b. clonidine

which of the following agents offers an intervention for the control of tremor and tachycardia associated with alcohol withdrawal? a. phenobarbital b. clonidine c. verapamil d. naltrexone

b. 2 wk supply of nortriptyline

which of the following drugs is likely to be the most dangerous when taken in an overdose? a. 4 wk supply of fluoxetine b. 2 wk supply of nortriptyline c. 3 wk supply of venlafaxine d. 3 day supply of diazepam

c. 38 y/o woman with fatigue and anhedonia for the past two years

which of the following is most consistent with the dx of dysthymia? a. 23 y/o man with a 2 month episode of depressed mood after a job loss b. 45 y/o woman with jitterness and difficulty initiating sleep for the past 6 months c. 38 y/o woman with fatigue and anhedonia for the past two years d. 15 y/o boy with a school adjustment problem and weekend marijuana use for the past yr


Set pelajaran terkait

Chapter 25: PrepU - Acquired Conditions and Congenital Abnormalities in the Newborn

View Set

Chapt. 24: Management of Patients with Chronic Pulmonary Disease

View Set

Evolve - Health disparities and health equity

View Set

ATI Pharmacology End of Chapter 26

View Set